Những câu hỏi liên quan
Mai Tiến Đỗ
Xem chi tiết
Trần Minh Hoàng
2 tháng 1 2021 lúc 19:39

3: \(P=\dfrac{x}{\left(x+y\right)+\left(x+z\right)}+\dfrac{y}{\left(y+z\right)+\left(y+x\right)}+\dfrac{z}{\left(z+x\right)+\left(z+y\right)}\le\dfrac{1}{4}\left(\dfrac{x}{x+y}+\dfrac{x}{x+z}\right)+\dfrac{1}{4}\left(\dfrac{y}{y+z}+\dfrac{y}{y+x}\right)+\dfrac{1}{4}\left(\dfrac{z}{z+x}+\dfrac{z}{z+y}\right)=\dfrac{3}{2}\).

Đẳng thức xảy ra khi x = y = x = \(\dfrac{1}{3}\).

Bình luận (0)
Hoàng Đức Khải
Xem chi tiết
Ngoc Nhi Tran
Xem chi tiết
huyền nguyễn
Xem chi tiết
Đinh Đức Hùng
Xem chi tiết
Huy Hoang
Xem chi tiết
Quang Trung
23 tháng 4 2021 lúc 15:35

Từ giả thiết ta có :

\(x+y+z=xyz\Leftrightarrow\frac{1}{xy}+\frac{1}{yz}+\frac{1}{zx}=1\)

ta có : \(Q=\frac{y+2}{x^2}+\frac{z+2}{y^2}+\frac{x+2}{z^2}\)

\(=\frac{\left(x+1\right)+\left(y+1\right)}{x^2}+\frac{\left(y+1\right)+\left(z+1\right)}{y^2}+\frac{\left(z+1\right)+\left(x+1\right)}{z^2}-\left(\frac{1}{x}+\frac{1}{y}+\frac{1}{z}\right)\)

\(=\left(x+1\right)\left(\frac{1}{z^2}+\frac{1}{x^2}\right)+\left(y+1\right)\left(\frac{1}{x^2}+\frac{1}{y^2}\right)+\left(z+1\right)\left(\frac{1}{y^2}+\frac{1}{z^2}\right)-\left(\frac{1}{x}+\frac{1}{y}+\frac{1}{z}\right)\)

\(\ge\frac{2\left(x+1\right)}{zx}+\frac{2\left(y+1\right)}{xy}+\frac{2\left(z+1\right)}{yz}-\left(\frac{1}{x}+\frac{1}{y}+\frac{1}{z}\right)\)

\(=2\left(\frac{1}{x}+\frac{1}{y}+\frac{1}{z}\right)+2\left(\frac{1}{xy}+\frac{1}{yz}+\frac{1}{zx}\right)-\left(\frac{1}{x}+\frac{1}{y}+\frac{1}{z}\right)\)

\(=\frac{1}{x}+\frac{1}{y}+\frac{1}{z}+2\)

Áp dụng bđt \(\left(a+b+c\right)^2\ge3\left(ab+bc+ca\right)\)

Dấu " = " xảy ra khi và chỉ khi a = b = c

Ta có \(\left(\frac{1}{x}+\frac{1}{y}+\frac{1}{z}\right)^2\ge3\left(\frac{1}{xy}+\frac{1}{yz}+\frac{1}{zx}\right)=3\)

\(\Rightarrow\frac{1}{x}+\frac{1}{y}+\frac{1}{z}\ge\sqrt{3}\)

Do đó : \(Q\ge\sqrt{3}+2\). Dấu " = " xảy ra 

\(\Leftrightarrow\hept{\begin{cases}\frac{1}{x}=\frac{1}{y}=\frac{1}{z}\\z+y+z=xyz\end{cases}\Leftrightarrow x=y=z=\sqrt{3}}\)

Vậy Min \(Q=\sqrt{3}+2\)khi \(x=y=z=\sqrt{3}\)

Bình luận (0)
 Khách vãng lai đã xóa
Hày Cưi
Xem chi tiết
Giao Khánh Linh
Xem chi tiết
Phạm Hồ Thanh Quang
Xem chi tiết
Nguyễn Thiều Công Thành
18 tháng 7 2017 lúc 15:23

<=>4(x+y)=5

ta có:

\(S+5=\frac{4}{x}+4x+\frac{1}{4y}+4y\ge2\sqrt{\frac{4}{x}.4x}+2\sqrt{\frac{1}{4y}.4y}=2.4+2=10\)

\(\Rightarrow S\ge5\)

Vậy Min S=5 khi x=1;y=1/4

Bình luận (0)